Límite clásico de la formulación integral de trayectoria de la mecánica cuántica

Es bien sabido que si S , entonces la ruta clásica domina la integral de ruta de Feynman. Pero, ¿hay algo que demuestre que si S , entonces la trayectoria de la partícula se aproximará a la trayectoria clásica?

el argumento mi X pag ( i S / ) oscila fuertemente cada vez que 0 en este caso sólo la trayectoria que satisface d S = 0 contribuye a la integral de trayectoria... las otras trayectorias se 'interfieren' entre sí
Esto solo es completamente cierto si agrega una pequeña parte imaginaria al tiempo, de modo que las rutas de acción altas se suprimen un poco exponencialmente. Esto da un corte que hace que las integrales de Feynman sean matemáticamente sensatas, y siempre se usa implícita o explícitamente cuando se hace una integral de trayectoria.
La trayectoria de la partícula no se aproximará exactamente a la trayectoria clásica, sino que las contribuciones de las trayectorias importantes en la suma no serán diferentes de observar simplemente la trayectoria clásica. Esto es sutilmente diferente, porque todavía se puede describir una distribución de probabilidad sobre el espacio de fase en el límite clásico de la mecánica cuántica, todavía queda algún principio de superposición residual, pero sin interferencia.
La pregunta no está clara, aunque las siguientes preguntas relacionadas: 19417 , 32112 , 32237 sugieren lo que se ha demostrado en el artículo .

Respuestas (1)

En realidad, existe un enfoque muy interesante seguido por E.Gozzi y sus alumnos para expresar la probabilidad de transición en Mecánica Clásica en términos de una integral de trayectoria. Actualmente está estudiando la relación entre MQ y CM utilizando este enfoque.

Puedes echar un vistazo a su sitio: http://www-dft.ts.infn.it/~gozzi/ , allí encontrarás algunas referencias interesantes.